Errors in Reasoning Questions - - Question 75

The government's proposed 8 percent cut in all subsidies to arts groups will be difficult for those groups to absorb....

kadipin25 December 28, 2018

Why E over D?

I was down to D & E and chose D instead. Explain Thanks!

Replies
Create a free account to read and take part in forum discussions.

Already have an account? log in

Ravi December 28, 2018

@kadipin25,

Happy to help. The stimulus tells us that the government's proposed 8 percent cut in all subsidies to arts groups will be difficult for those groups to absorb. Then, the argument says that based on their response to last year's cut, the proposed 8 percent cut will not put arts groups out of business.

This sounds like the conclusion. But why does the author think this? What's the support?

The argument then states that last year there was also an 8 percent cut, and though private fundraising was very difficult for the arts groups in the current recessionary economy, they did survive. This is the premise. O.K., so because there was also an 8 percent cut to the subsidies last year and the arts groups were able to privately fundraise to offset the subsidy cuts, the new 8 percent cut this year will not put them out of existence. But what if the arts groups barely survived after last year's cuts? Couldn't these new cuts put their economic health in the toilet and make them cease to exist? This is the problem with this argument.

The question stem asks us to find the answer choice that shows why the reasoning in the argument is flawed.

Answer A is incorrect because whether or not the economy will improve has no bearing on the argument. The only mention of the economy was that the arts groups privately fundraised during a difficult recessionary economy. However, the argument does not assume that the economy will improve.

Answer B is incorrect because we do not care about whether or not there "should" be government subsidies to arts groups. Opinions do not matter for us. We can eliminate this answer.

Answer C is incorrect because there is no equating of survival with flourishing. The author does not mention flourishing anywhere in her argument. We can eliminate this answer.

Answer D is incorrect because it is not true. The author could very well be taking into account that the dollar amount of the proposed cut is lower than the dollar amount of last year's cut. What the author is failing to do is to realize that the cumulative effect of these cuts may be enough to put the arts groups out of existence. D does not describe this, so it is not our answer.

Answer E is correct because it paraphrases the flaw we identified when we were analyzing the stimulus. The author is assuming that the cumulative effect of these cuts will not be enough to put the arts groups out of existence, but this isn't necessarily true.

For example, let's say that last year's subsidies cut was from a base of $100. An 8% cut on $100 would give us $92. If this year's cut is another 8% lower than $92, then we would be left with $84.64. Let's say that the operating budget of the arts groups is $95. With only $92 in subsidies, the arts groups would need to fundraise for $3 to make up the difference. Although it was hard to do last year, let's say they came up with $3, giving them a total of $95 to cover their costs.

This year, on the other hand, the arts groups only have $84.64 in subsidies, and their operating costs are still $95. If they were to fundraise $3 again this year, they'd only have $87.64 total, which is $7.36 shy of their $95 in operating costs. This hypothetical scenario shows how the cumulative effects of these cuts could put the arts groups out of business.

Hope this helps. Let us know if you have any more questions!

kadipin25 December 29, 2018

Ok Thank you

Ravi December 31, 2018

@kadipin25, you're welcome! Happy to help!